next up previous
Next: Solution to Problem 3. Up: No Title Previous: Solution to Problem 1.

Solution to Problem 2.

2. [30 points] Suppose tex2html_wrap_inline598 , tex2html_wrap_inline600 , tex2html_wrap_inline602 are i.i.d. random variables with the exponential distribution having mean tex2html_wrap_inline688 . That is, the density is given by

  equation240

Let

  equation249

denote the sample mean. Of course, the ``population'' variance is

  equation256

Show that for some tex2html_wrap_inline690 ,

  equation260

and determine the value of tex2html_wrap_inline690 .

Solution: We will apply the tex2html_wrap_inline694 -method: If tex2html_wrap_inline696 tex2html_wrap_inline698 tex2html_wrap_inline700 where tex2html_wrap_inline702 and if h is a function differentiable at tex2html_wrap_inline688 , then tex2html_wrap_inline708 tex2html_wrap_inline698 tex2html_wrap_inline712 . Of course,

  equation268

So, applying the tex2html_wrap_inline694 -method with tex2html_wrap_inline716 , we have tex2html_wrap_inline718 = tex2html_wrap_inline722 and

  equation274

i.e., tex2html_wrap_inline690 in (15) is tex2html_wrap_inline726 . (NOTE: We know that it would have to be a 4'th power of tex2html_wrap_inline688 just by dimensional analysis.)



Dennis Cox
Tue Nov 11 20:55:50 CST 1997